LSAT and Law School Admissions Forum

Get expert LSAT preparation and law school admissions advice from PowerScore Test Preparation.

User avatar
 Dave Killoran
PowerScore Staff
  • PowerScore Staff
  • Posts: 5852
  • Joined: Mar 25, 2011
|
#88287
Complete Question Explanation
(The complete setup for this game can be found here: lsat/viewtopic.php?f=176&p=88283#p88283)

The correct answer choice is (A).

The condition in the question stem creates a V :longline: Y :longline: X sequence. From the fourth rule we can add in Z, creating a Z :longline: V :longline: Y :longline: X sequence. As these are the only four unplaced variables, we can determine that these four must be questioned on days one, two, four, and five:

G3-Q14-d1.png

Answer choice (A) is the correct answer. As shown above, V is questioned second, and the confession status of the suspect questioned on day two is not determined.

Answer choice (B) is incorrect because Y is questioned on day four, and the suspect questioned on day four does not confess.

Answer choice (C) is incorrect because X is questioned on day five, and the suspect questioned on day five confesses.

Answer choice (D) is incorrect because X is questioned on day five.

Answer choice (E) is incorrect because Z is questioned on day one.
You do not have the required permissions to view the files attached to this post.
 deck1134
  • Posts: 160
  • Joined: Jun 11, 2018
|
#47647
Dear PowerScore Staff,

I cannot seem to get this one either.

If Y was after V but before X,

I put:
_ _ _ N C C N
Z X/V T X/V _ W S

Why does this mandate that V did not confess? Isn't it also true that X did not confess?
 Adam Tyson
PowerScore Staff
  • PowerScore Staff
  • Posts: 5153
  • Joined: Apr 14, 2011
|
#47700
Your diagram doesn't conform to the local restrictions in the question, deck, in that you 1) haven't placed Y anywhere and 2) you have V and X potentially swapping places, but the question requires that V come before Y, which comes before X, giving us a V-Y-X order. Once you fix that, you should get this order:

ZVTYXWS

Thus, answer A could be true, because the second suspect has the option of confessing or not! Note that this is a COULD be true question, not a MUST be true question, so this local scenario doesn't mandate that V confess, but rather allows V to do so. That's what makes A the correct answer - it could be true.

Answer B: Y cannot confess, because he is questioned 4th and the 4th suspect does not confess, so this answer choice cannot be true

Answer C: X must confess, because he is questioned 5th and the 5th and 6th suspects must both confess, so this answer choice cannot be true

Answer D: X is questioned 5th, not 4th, so this answer choice cannot be true

Answer E: Z is questioned first, not 2nd, so this answer choice cannot be true

Be sure on these local questions to draw out the "if" portion of the question fully, making additional inferences as you do so!

Get the most out of your LSAT Prep Plus subscription.

Analyze and track your performance with our Testing and Analytics Package.